8
$\begingroup$

Let $k$ be a field and $n$ a nonnegative integer. For any matrix $U\in\mathrm{M}_n\left(k\right)$, let $\mathrm{ad} U$ denote the map $\mathrm{M}_n\left(k\right)\to \mathrm{M}_n\left(k\right),\ V\mapsto UV-VU$. Thus, $\mathrm{ad} U$ is an element of the $k$-algebra $\mathrm{End}_k\left(\mathrm{M}_n\left(k\right)\right)$.

Is it true that for every $n\times n$-matrix $A$ over $k$, and for every $m\in\mathbb N$, the endomorphism $\mathrm{ad}\left(A^m\right)$ can be written in the form $P\left(\mathrm{ad}A\right)$ for some polynomial $P\in k\left[X\right]$ satisfying $P\left(0\right)=0$ ?

I know that this holds when $A$ is diagonalizable, and in that case it is used in the proof of Cartan's Lemma from Lie algebra theory. If it holds generally and can be proven neatly, it could be used to tidy up the proof of Cartan's Lemma (which, in the form I know it, is rather ugly, requiring an algebraic extension of the ground field and the use of Jordan's normal form).

$\endgroup$
4
  • $\begingroup$ To check whether it is true, I would try to use the Jordan normal form of a matrix $\endgroup$
    – user24527
    Jul 1, 2012 at 5:51
  • $\begingroup$ NN: Unfortunately this isn't a fact which clearly holds for a matrix if it holds for each Jordan block. $\endgroup$ Jul 1, 2012 at 8:42
  • $\begingroup$ is it on purpose that the power ($n$ in $ad(A^n)$) coincides with the size $n$ of the matrix? $\endgroup$
    – YCor
    Jul 1, 2012 at 10:07
  • $\begingroup$ Oops! No! Thanks for poinitng out the typo. $\endgroup$ Jul 1, 2012 at 10:08

1 Answer 1

7
$\begingroup$

I don't think it's true even when $A$ is diagonalisable.

Suppose that $A$ is a diagonal matrix, with $(i,i)$th entry $\lambda_i$; thus $A = \rm{diag}$$(\lambda_i)$. Then we can write $\rm{ad}(A) = \rm{diag}$$(\lambda_i - \lambda_j)$, so

$P(\rm{ad}$$(A)) = \rm{diag}$$ (P (\lambda_i - \lambda_j) )$

for any polynomial $P(t)$.

On the other hand, $A^m = \rm{diag}$$(\lambda_i^m)$ so

$\rm{ad}$$(A^m) = \rm{diag}$$(\lambda_i^m - \lambda_j^m)$.

So if $P(\rm{ad}$$(A)) = $$\rm{ad}$$(A^m)$ then $P(\lambda_i - \lambda_j) = \lambda_i^m -\lambda_j^m$ for all $i,j$ such that $1\leq i,j\leq n$.

But it can happen that $\lambda_i - \lambda_j = \lambda_a - \lambda_b$ for two pairs of indices $(i,j)$ and $(a,b)$ with $\lambda_i^m - \lambda_j^m \neq \lambda_a^m - \lambda_b^m$.

For a concrete example we can take $A = \rm{diag}$$(2,1,4,3)$.

$\endgroup$
1
  • $\begingroup$ Sorry! My argument was completely wrong. $\endgroup$ Jul 1, 2012 at 10:38

Your Answer

By clicking “Post Your Answer”, you agree to our terms of service and acknowledge you have read our privacy policy.

Not the answer you're looking for? Browse other questions tagged or ask your own question.